Difference between revisions of "2017 AMC 10B Problems/Problem 5"

m (updating the box at the bottom)
Line 7: Line 7:
 
==Solution==
 
==Solution==
  
{{AMC10 box|year=2017|ab=B|num-b=2|num-a=4}}
+
{{AMC10 box|year=2017|ab=B|num-b=4|num-a=6}}
 
{{MAA Notice}}
 
{{MAA Notice}}

Revision as of 11:35, 16 February 2017

Problem

[QUESTION]

$\textbf{(A)}\ [???]\qquad\textbf{(B)}\ [???]\qquad\textbf{(C)}\ [???]\qquad\textbf{(D)}\ [???]\qquad\textbf{(E)}\ [???]$

Solution

2017 AMC 10B (ProblemsAnswer KeyResources)
Preceded by
Problem 4
Followed by
Problem 6
1 2 3 4 5 6 7 8 9 10 11 12 13 14 15 16 17 18 19 20 21 22 23 24 25
All AMC 10 Problems and Solutions

The problems on this page are copyrighted by the Mathematical Association of America's American Mathematics Competitions. AMC logo.png